K
Khách

Hãy nhập câu hỏi của bạn vào đây, nếu là tài khoản VIP, bạn sẽ được ưu tiên trả lời.

4 tháng 4 2020
https://i.imgur.com/zbuOFBx.jpg

cảm ơn bn nhiều

25 tháng 7 2020

Câu 1:
\(4\sqrt[4]{\left(a+1\right)\left(b+4\right)\left(c-2\right)\left(d-3\right)}\le a+1+b+4+c-2+d-3=a+b+c+d\)

Dấu = xảy ra khi a = -1; b = -4; c = 2; d= 3

25 tháng 7 2020

\(\frac{a^2}{b^5}+\frac{1}{a^2b}\ge\frac{2}{b^3}\)\(\Leftrightarrow\)\(\frac{a^2}{b^5}\ge\frac{2}{b^3}-\frac{1}{a^2b}\)

\(\frac{2}{a^3}+\frac{1}{b^3}\ge\frac{3}{a^2b}\)\(\Leftrightarrow\)\(\frac{1}{a^2b}\le\frac{2}{3a^3}+\frac{1}{3b^3}\)

\(\Rightarrow\)\(\Sigma\frac{a^2}{b^5}\ge\Sigma\left(\frac{5}{3b^3}-\frac{2}{3a^3}\right)=\frac{1}{a^3}+\frac{1}{b^3}+\frac{1}{c^3}+\frac{1}{d^3}\)

18 tháng 8 2019

By Cauchy-Schwarz, we have:

\(VT\ge\frac{\left(a^3+b^3+c^3\right)^2}{2\left(a^3+b^3+c^3\right)+a^2b+b^2c+c^2a}\)

We will prove: \(a^2b+b^2c+c^2a\le a^3+b^3+c^3\)

\(\Leftrightarrow a^2b+b^2c+c^2a+3abc\le a^3+b^3+c^3+3abc\)

By Schur, we have: \(RHS\ge ab\left(a+b\right)+bc\left(b+c\right)+ca\left(a\right)\)

So we're only need to prove: \(ab\left(a+b\right)+bc\left(b+c\right)+ca\left(c+a\right)\ge a^2b+b^2c+c^2a+3abc\)

\(\Leftrightarrow ab^2+bc^2+ca^2\ge3abc\)

It is true by AM-GM ineq', so we have Q.E.D.

P/s: Em thử giải bài này bằng tiếng Anh (để tự luyện kĩ năng tiếng anh, tí em giải lại theo tiếng việt)

18 tháng 8 2019

Ấy nhầm:V

By Schur, we have \(RHS\ge ab\left(a+b\right)+bc\left(b+c\right)+ca\left(c+a\right)\)

So we're only need to prove \(ab\left(a+b\right)+bc\left(b+c\right)+ca\left(c+a\right)\ge a^2b+b^2c+c^2a\)

Còn lại y chang:v

23 tháng 11 2017

chứng minh \(\sqrt{2x+1}\)là số vô tỉ

NV
6 tháng 5 2021

Ta chứng minh BĐT sau với các số dương:

\(\dfrac{1}{x}+\dfrac{1}{y}\ge\dfrac{4}{x+y}\)

Thật vậy, BĐT tương đương: \(\dfrac{x+y}{xy}\ge\dfrac{4}{x+y}\Leftrightarrow\left(x+y\right)^2\ge4xy\)

\(\Leftrightarrow x^2-2xy+y^2\ge0\Leftrightarrow\left(x-y\right)^2\ge0\) (luôn đúng)

Áp dụng:

\(\dfrac{1}{a}+\dfrac{1}{b}\ge\dfrac{4}{a+b}\) ; \(\dfrac{1}{b}+\dfrac{1}{c}\ge\dfrac{4}{b+c}\) ; \(\dfrac{1}{c}+\dfrac{1}{a}\ge\dfrac{4}{c+a}\)

Cộng vế với vế:

\(2\left(\dfrac{1}{a}+\dfrac{1}{b}+\dfrac{1}{c}\right)\ge\dfrac{4}{a+b}+\dfrac{4}{b+c}+\dfrac{4}{c+a}\)

\(\Leftrightarrow\dfrac{1}{a}+\dfrac{1}{b}+\dfrac{1}{c}\ge\dfrac{2}{a+b}+\dfrac{2}{b+c}+\dfrac{2}{c+a}\)

NV
6 tháng 5 2021

b.

Ta có:

\(\dfrac{1}{a}+\dfrac{1}{b}\ge\dfrac{4}{a+b}\Rightarrow\dfrac{3}{a}+\dfrac{3}{b}\ge\dfrac{12}{a+b}\) (1)

\(\dfrac{1}{b}+\dfrac{1}{c}\ge\dfrac{4}{b+c}\Rightarrow\dfrac{2}{b}+\dfrac{2}{c}\ge\dfrac{8}{b+c}\) (2)

\(\dfrac{1}{c}+\dfrac{1}{a}\ge\dfrac{4}{c+a}\) (3)

Cộng vế với vế (1); (2) và (3):

\(\dfrac{4}{a}+\dfrac{5}{b}+\dfrac{3}{c}\ge4\left(\dfrac{3}{a+b}+\dfrac{2}{b+c}+\dfrac{1}{c+a}\right)\) (đpcm)

Dấu "=" xảy ra khi \(a=b=c\)

7 tháng 8 2019

Áp dụng BĐT cosi ta có

\(\frac{1}{a^3}+\frac{1}{a^3}+\frac{1}{b^3}\ge\frac{3}{a^2b}\)\(\frac{1}{b^3}+\frac{1}{b^3}+\frac{1}{c^3}\ge\frac{3}{b^2c}\)\(\frac{1}{c^3}+\frac{1}{c^3}+\frac{1}{d^3}\ge\frac{3}{c^2d}\)

\(\frac{1}{d^3}+\frac{1}{d^3}+\frac{1}{a^3}\ge\frac{3}{d^2a}\)

Cộng các BĐt trên ta có 

\(\frac{1}{a^3}+\frac{1}{b^3}+\frac{1}{c^3}+\frac{1}{d^3}\ge\frac{1}{a^2b}+\frac{1}{b^2c}+\frac{1}{c^2d}+\frac{1}{d^2a}\)(1)

Áp dụng BĐT buniacoxki ta có

\(\left(\frac{a^2}{b^5}+\frac{b^2}{c^5}+\frac{c^2}{d^5}+\frac{d^2}{a^5}\right)\left(\frac{1}{a^2b}+\frac{1}{b^2c}+\frac{1}{c^2d}+\frac{1}{d^2a}\right)\ge \left(\frac{1}{a^3}+\frac{1}{b^3}+\frac{1}{c^3}+\frac{1}{d^3}\right)^2\)

Kết hợp với (1)  ta được ĐPCM

Dấu bằng xảy ra khi a=b=c

9 tháng 7 2017

Lần sau đăng ít 1 thôi đăng nhiều ngại làm, bn đăng nhiều nên tui hướng dẫn sơ qua thôi tự làm đầy đủ vào vở

Bài 1:

Áp dụng BĐT AM-GM ta có:

\(a^4+b^4\ge2a^2b^2;b^4+c^4\ge2b^2c^2;c^4+a^4\ge2c^2a^2\)

Cộng theo vế 3 BĐT trên rồi thu gọn

\(a^4+b^4+c^4\ge a^2b^2+b^2c^2+c^2a^2\)

Áp dụng tiếp BĐT AM-GM

\(a^2b^2+b^2c^2=b^2\left(a^2+c^2\right)\ge2b^2ac\)

Tương tự rồi cộng theo vế có ĐPCM

Bài 2:

Quy đồng  BĐT trên ta có:

\(\frac{a^2}{b^2}+\frac{b^2}{a^2}-\frac{a}{b}-\frac{b}{a}\ge0\)

\(\Leftrightarrow\frac{\left(a-b\right)^2\left(a^2+ab+b^2\right)}{a^2b^2}\ge0\) (luôn đúng)

Bài 4: Áp dụng BĐT AM-GM 

\(a^3+b^3=\left(a+b\right)\left(a^2-ab+b^2\right)\)

\(\ge\left(a+b\right)\left(2ab-ab\right)=ab\left(a+b\right)\)

\(\Rightarrow\frac{a^3+b^3}{ab}\ge\frac{ab\left(a+b\right)}{ab}=a+b\)

Tương tự rồi cộng theo vế

Bài 5: sai đề tự nhien có dấu - :v nghĩ là +

9 tháng 7 2017

ai k mình k lại [ chỉ 3 người đầu tiên mà trên 10 điểm hỏi đáp ]

 
15 tháng 2 2022

Sử dụng bất đẳng thức Bunhiacopxki dạng phân thức và khi đó ta được:

\(\frac{a^5}{a^2+ab+b^2}+\frac{b^5}{b^2+bc+c^2}+\frac{c^5}{c^2+ca+a^2}\ge\)

\(\frac{\left(a^3+b^3+c^3\right)^2}{a^3+b^3+c^3+a^2b+ab^2+b^2c+bc^2+c^2a+ca^2}\)

\(\Rightarrow\)Ta cần chỉ ra được:

\(\frac{\left(a^3+b^3+c^3\right)^2}{a^3+b^3+c^3+a^2b+ab^2+b^2c+bc^2+c^2a+ca^2}\ge\frac{a^3+b^3+c^3}{3}\)

Hay: \(2\left(a^3+b^3+c^3\right)\ge a^2b+ab^2+b^2c+bc^2+c^2a+ca^2\)

Dễ thấy: \(a^3+b^3\ge ab\left(a+b\right);b^3+c^3\ge bc\left(b+c\right);c^3+a^3\ge ca\left(c+a\right)\)

Cộng theo vế các bất đẳng thức trên ta được:

\(2\left(a^3+b^3+c^3\right)\ge a^2b+ab^2+b^2c+bc^2+c^2a+ca^2\)

Vậy bất đẳng thức đã được chứng minh.